0% found this document useful (0 votes)
18 views4 pages

Analysis I MS Sol 2008-09

Copyright
© © All Rights Reserved
We take content rights seriously. If you suspect this is your content, claim it here.
Available Formats
Download as PDF, TXT or read online on Scribd
0% found this document useful (0 votes)
18 views4 pages

Analysis I MS Sol 2008-09

Copyright
© © All Rights Reserved
We take content rights seriously. If you suspect this is your content, claim it here.
Available Formats
Download as PDF, TXT or read online on Scribd
You are on page 1/ 4

Analysis 1 - Mid-Sem exam - 2008-09

B.Math (Hons.)

Problem 1. Show that the ordered field R of real numbers has the Archimedean property.

Solution. We need to prove that for every x > 0, y > 0 , there exists a natural number n such that
nx > y. Let N denote the set of natural numbers. We first prove the following lemma.

Lemma: N is unbounded.

Suppose, N is bounded. By, l.u.b axiom there exists a least upper bound α such that n ≤ α for all n ∈ N.
Now, since α is the least upper bound, α − 1 is not an upper bound. Thus, there exists a k such that
k ≥ α − 1. This implies that k + 2 > α. Hence we get a contradiction. Thus, N is unbounded.

Now, given x > 0, y > 0 consider y/x. By the previous lemma, there exists a n ∈ N such that n > y/x.
Thus we have nx > y.

Problem 2. For any two subsets A, B of R , define

A+B = {a + b : a ∈ A, b ∈ B};
AB = {ab : a ∈ A, b ∈ B}

Show that if A, B are bounded above,

sup(A + B) = sup(A) + sup(B).

Give an example to show that


sup(AB) = sup(A).sup(B),
need not be true.

Solution. (a) To prove that sup(A + B) = sup(A) + sup(B).

Since, A and B are bounded, sup(A), sup(B), and sup(A + B) are finite.

claim 1: sup(A + B) ≤ sup(A) + sup(B).

Let x ∈ A + B be any arbitrary element. Then x = a + b for some a ∈ A and b ∈ B. And we know that
a ≤ sup(A) and b ≤ sup(B). Hence x = a + b ≤ sup(A) + sup(B).

Hence x ≤ sup(A) + sup(B) for all x ∈ A + B. Thus we have sup(A + B) ≤ sup(A) + sup(B).

claim 2: sup(A + B) ≥ sup(A) + sup(B).

Let  > 0 be arbitrary. Then there exists an a ∈ A such that a ≥ sup(A) − /2. Similarly, there exists a
b ∈ B such that b ≥ sup(B) − /2. Then a + b ≥ sup(A) + sup(B) − .

But we have sup(A + B) ≥ a + b. Hence for all  > 0 we have sup(A + B) ≥ sup(A) + sup(B) − .
Therefore we have sup(A + B) ≥ sup(A) + sup(B).

1
Thus, from claim 1 and claim 2, we have sup(A + B) = sup(A) + sup(B).

(b) To show that sup(AB) = sup(A)sup(B) is not necessarily true.

Let A = [−1, 0] and B = [−1, 0]. We have sup(A) = sup(B) = 0, and hence sup(A).sup(B) = 0.

Now, we have that 1 ∈ A.B. Thus sup(A, B) ≥ 1. Hence we cannot have sup(AB) = sup(A)sup(B).

Problem 3. Show that intervals [0, 1] and (0, 1) have same cardinality.

Solution. Let  
1
H= :n∈N
n
Define the function f : (0, 1) → [0, 1] such that


0 if x = 1/2

1 if x = 1/3
f (x) =


1/(n − 2) if x = 1/n ; n ≥ 3 and x ∈ N
x if x ∈
/H

Now, define g : [0, 1] → (0, 1) as



1/2 if x=0

1/3 if x=1
f (x) =


1/(n + 2) if x = 1/n ; n ≥ 3 and x ∈ N
x if x∈/ H ∪ {0, 1}

Now note that f ◦ g = I and g ◦ f = I. Hence we have that f is a bijection, and the intervals [0, 1] and
(0, 1) have same cardinality.

Problem 4. Let {an } be a sequence of real numbers converging to a ∈ R. Define


1
bn = (a1 + a2 + · · · + an )
n
for n ≥ 1. Show that {bn } is a converging sequence converging to a.

Solution. Let  > 0 be arbitrary. Since {ai } is converging sequence, we can choose a K such that

| ai − a |< ∀i ≥ K
2
Let
K
X
M= | ai − a |
i=1

2
For n ≥ K, we have
n
1 X
| bn − a | = ai − na
n i=1
n
1X
≤ | ai − a |
n i=1
K n
1X 1 X
= | ai − a | + | ai − a |
n i=1 n
i=K+1
M n−K
≤ +
n 2 n
M 
≤ +
n 2

Now there exists a N such that M/n < /2 for all n ≥ N . Therefore for all n ≥ max{N.K}, we have
 
| bn − a |≤ + ≤ .
2 2
Since  is arbitrary bn converges to a.
Problem 5. Show that every Cauchy sequence of real numbers is convergent.

Solution. Let {xn } be a cauchy sequence in R. Let  > 0 be arbitrary. There exist N > 0, such that for
all m, n ≥ N , we have that |xn − xm | < /2. In particular we have |xn − xN | < /2. Or equivalently,
xn ∈ (xN − /2, xN + /2) ∀n ≥ N (1)

From this we make the following observations:

(a) For all n > N we have xn > xN − /2


(b) If xn > xN + /2, then n ∈ {1, 2, · · · N − 1}. Thus, the set of n such that xn ≥ xN + /2 is finite.

Let S = {x ∈ R : there exists infintely many n such thatxn ≥ x}. We claim that S is non-empty,
bounded above and that sup(S) is the limit of the given sequence.

Let  = 1. There there exists a N such that (1) holds. Therefore xN − 1/2 belongs to S. Hence S is non
empty.

We claim that xN + 1 is an upper bound for S. If this were not true, there exists a y ∈ S such that
y > xN + 1 and xn ≥ y for infinitely many n. This implies that xn > xN + 1 for infinitely many n. This
results in a contradiction from (b). Hence xN + 1 is an upper bound for S.

By the LUB axiom, there exists a l ∈ R, such that sup(S) = l. We claim that lim xn = l. Let  > 0 be
given. As l is an upper bound for S and xN − /2 ∈ S, from above, we infer that xN − /2 ≤ l. Since
l is the least upperbound for S and xN + /2 is an upper bound for S, we see that l ≤ xN + /2. Thus
we have xN − /2 ≤ l ≤ xN + /2 or
|xN − l| ≤ /2
For n ≥ N , we have
|xn − l| ≤ |xn − xN | + |xN − l|
≤ /2 + /2
Thus we have shown that lim xn = l.

3
Problem 6. Let A be a subset of R. Show that the interior of A is the largest open set contained in A.

Solution. Denote interior of A as int(A). To show that, int(A) is the largest open set contained in A.

Firstly, by definition of int(A), we have that int(A) is an open set contained in A. Let X be the largest
open set. We have that X ⊇ int(A).

Let x ∈ X. Since X is open, there exists an  > 0 such that B(x, ) ⊂ X ⊂ A. From the definition of
int(A) we have X ⊆ int(A).

Hence X = int(A).

Problem 7. Suppose g : R → R is a continuous function. Define a new function g+ : R → R by


(
0 if g(x) < 0,
g+ (x) =
x otherwise.

Show that if g is continuous then g+ is continuous. Give an example where g+ is continuous but g is not
continuous.

Solution. (a) To show that g+ is continuous. Firstly, note that

g(x) | g(x) |
g+ (x) = +
2 2

claim 1 : Let x ∈ R. If g is continuous at x then | g | is continuous at x.

Let  > 0 be arbitrary. Since, g is continuous at x there exists a δ > 0 such that if | x − y |< δ, then

| g(x) − g(y) |< 

Now,
| g(x) | − | g(y) | ≤| g(x) − g(y) |≤ 

thus we have that at x, there exists a δ > 0 such that if | x − y |< δ, then

| g(x) | − | g(y) | < 

Thus, | g | is continuous.

Since, linear combination of continuous functions is continuous, we have g+ is continuous.

(b) To show that if g+ is continuous, g need not be continuous.

Let g : R → R, given by (
0 if x ∈ Q
g(x) =
−1 if x ∈ Qc
Note that g is discontinuous. Now, we have g+ (x) = 0 for all x ∈ R, which is continuous. Hence if g+ is
continuous, g need not be continuous.

You might also like